Integration – Question on Beta Function

gamma functionhypergeometric functionintegrationsequences-and-seriesspecial functions

I need an asymptotic expansion/closed form for $$\sum_{k=1}^{\infty}\int_{0}^{\infty}(B(x+n+k,n+1))^2\ dx$$ where $B(m,n)$ is the Beta function and $n\in\mathbb{N}$.

Denote $$I_n=\sum_{k=1}^{\infty}\int_{0}^{\infty}(B(x+n+k,n+1))^2\ dx$$ By definition of Beta function
$$I_n=\sum_{k=1}^{\infty}\int_{0}^{\infty}\left(\frac{\Gamma(x+n+k)\Gamma(n+1)}{\Gamma(x+2n+k+1)}\right)^2\ dx$$ $$I_n=(n!)^2\sum_{k=1}^{\infty}\int_{0}^{\infty}\left(\frac{\Gamma(x+n+k)}{\Gamma(x+2n+k+1)}\right)^2\ dx$$ Now interchanging the summation and integral (which needs to be justified) we get $$I_n=(n!)^2\int_{0}^{\infty}\sum_{k=1}^{\infty}\left(\frac{\Gamma(x+n+k)}{\Gamma(x+2n+k+1)}\right)^2\ dx$$
Now the infinite sum above is see here

$$I_n=(n!)^2\int_{0}^{\infty} \left(\frac{\Gamma(x+n+1)}{\Gamma(x+2n+2)}\right)^2{}_3F_2\left ( 1,x+n+1,x+n+1;x+2n+2,x+2n+2;1 \right)dx$$

where ${}_3F_2$ represents the hypergeometric function.

Thank you!

Edit: @Gary and @TymaGaidash have in their elegant answers found that $$\sum_{k=1}^{\infty}\int_{0}^{\infty}\operatorname B^2(x+n+k,n+1)\ dx =\binom{2n}n\gamma-\sum_{j=0}^n\binom nj^2(2(H_{n-j}-H_j)\ln((j+n)!)+H_{n+j})$$ So if we define$$a_n:= \sum_{j=0}^n\binom nj^2(2(H_{n-j}-H_j)\ln((j+n)!) $$ I need an asymptotic expansion for $a_n$ as $n\to \infty$. I believe that we can apply Laplace's method of asymptotic expansion of integrals (see here p.$322$)

Question:
I need to prove that the limit $$ \lim_{n\to \infty} \frac{n\ 4^{2n}}{e^{2n}}\{-d_{2n} a_n\}\leq \frac{3}{4}$$ where $d_{2n}=\text{LCM}(1,2,…,2n)$, $\{x\}$ is the fractional part of $x$ and $a_n$ is defined as above.

Edit Can we at least simplify the above fractional part $\{-d_{2n} a_n\}$? Can we use the (anti) symmetry in $a_n$ to get a simpler form of $\{-d_{2n} a_n\}$?

I would really appreciate an answer which I can accept. Thank you!

Best Answer

This is not a complete answer. Using the limit representation in Tyma Gaidash's answer, and the known results $$ H_{n + j + r} = \log (n + j + r) + \gamma + o(1), \quad r\to+\infty, $$ (with $\gamma$ being the Euler–Mascheroni constant) and $$ \sum\limits_{j = 0}^n {\binom{n}{j}^2 } = \binom{2n}{n}, $$ we can derive the explicit formula $$ \boxed{I_n = \binom{2n}{n}\gamma - \sum\limits_{j = 0}^n {\binom{n}{j}^2 (2(H_{n - j} - H_j )\log (( n+j)!) + H_{n + j} )} ,} $$ provided that $$ \mathop {\lim }\limits_{r \to + \infty } \sum\limits_{j = 0}^n {\binom{n}{j}^2 \left( {2(H_{n - j} - H_j )\log ((n + j + r)!) + \log (n + j + r)} \right)} = 0. $$ Denote the expression under the limit by $S_n(r)$. Performing the change of summation index $j\to n-j$ and taking the average with the original expression, we find $$ S_n (r) = \sum\limits_{j = 0}^n {\binom{n}{j}^2 \left( {(H_{n - j} - H_j )\log \frac{{(n + j + r)!}}{{(2n - j + r)!}} + \frac{1}{2}\log ((2n - j + r)(n + j + r))} \right)} . $$ Stirling's formula and the Maclaurin series of the logarithm then yields $$ S_n (r) = (\log r)\sum\limits_{j = 0}^n {\binom{n}{j}^2 \left( {(H_{n - j} - H_j )(2j - n) + 1} \right)} + o(1) $$ as $r\to+\infty$. Therefore, it remains to show that $$ \sum\limits_{j = 0}^n {\binom{n}{j}^2 ((H_{n - j} - H_j )(2j - n) + 1)} =0 $$ for all $n\ge 1$. Because of the symmetry in $H_{n - j} - H_j$, this may be further simplifed to the claim that $$ \sum\limits_{j = 0}^n {\binom{n}{j}^2 (2j(H_{n - j} - H_j )+ 1)} =0 $$ for all $n\ge 1$. Computer algebra software confirms this for $n=1,2,3\ldots,200$.

Addendum. The limit expression in the original version of Tyma Gaidash's answer was $$ I_n=\lim_{r\to\infty}\sum_{j=0}^n\binom n j^2\left(2 (H_{n-j}-H_j)\ln\left(\frac{(n+j+r)!}{(n+j)!}\right)+H_{n+j+r}-H_{n+j}\right). $$

Related Question